LSAT and Law School Admissions Forum

Get expert LSAT preparation and law school admissions advice from PowerScore Test Preparation.

 Administrator
PowerScore Staff
  • PowerScore Staff
  • Posts: 8917
  • Joined: Feb 02, 2011
|
#36657
Complete Question Explanation

Assumption—CE. The correct answer choice is (A)

Proponents of intellectual property rights, such as the right to patent new inventions, argue that the
best way to ensure continued innovation is to maximize the property holders’ (patent holders’) right to
benefit financially from their developments. The stimulus takes this argument one step further and makes
the extreme claim that original development will cease altogether if there is no financial incentive for
inventing new products. For this conclusion to be possible, it must be assumed that nothing other than
financial incentive could motivate original development and invention. Answer choice (A) is correct
because it shows that financial incentive is singularly necessary.

Answer choice (A): This is the correct answer choice. As with any correct answer choice to an
Assumption question, this answer choice must pass the Assumption Negation Test. Once negated, the
correct answer choice will invalidate the argument. For answer choice (A), if the stimulus is negated
it states that financial reward is not the only effective incentive in motivating people to develop new
inventions, and the conclusion of the stimulus is no longer correct. That is, some people will continue to
engage in original development even if patent rights are not granted because these people will not need a
financial incentive to produce new inventions. Because answer choice (A) shows the absolute necessity
of financial incentive, it is the correct answer.

Answer choice (B): Some proponents of intellectual property rights are concerned about the relationship
between inventors and manufacturers. But the argument in the stimulus is not contingent upon the nature
of that relationship. The conclusion is that without patent rights and the financial incentives that they
provide no new inventions will be developed. Thus, the conclusion does not depend upon the assumption
that inventors make more total profit than manufacturers.

Answer choice (C): This answer choice suggests that the economic benefit to an inventor of obtaining
patent rights must be significantly greater than the associated cost. Presumably, this is necessary to
justify the investment of time and energy in obtaining a patent. But the conclusion is drawn from the
complete absence of financial incentive. Knowledge about the cost of applying for patent rights does not
allow for an evaluation of whether the lack of financial incentive would cause original development to
cease.

Answer choice (D): The stimulus is not dependent upon the restriction of patent rights to those products
which are sufficiently unique. Very similar products may provide enough financial incentive to promote
continued innovation and the stimulus only predicts what will occur if there is no financial incentive
present.

Answer choice (E): The duration of patent rights and the development cost of new inventions are not
directly linked to the financial incentive mentioned in the stimulus. Therefore, the relationship between
these two factors need not be known in order to successfully draw the conclusion. Furthermore, if this
answer choice is negated and the length of patent rights for inventions that were inexpensive to develop
was longer than the length of patent rights for expensive inventions, the conclusion would be unaffected.

Get the most out of your LSAT Prep Plus subscription.

Analyze and track your performance with our Testing and Analytics Package.